The Unapologetic Mathematician

Mathematics for the interested outsider

Non-Lebesgue Measurable Sets

I need to make up for missing a post earlier this week…

The most important observation about the fact that Lebesgue measurable sets might not be all of P(X) is sort of tautological: it means that there may be subsets of the real line which are not Lebesgue measurable. That is, sets for which it is impossible to give a sense of “how much space they take up”, in a way compatible with the length of an interval.

And we can show that such sets do, in fact, exist. At least, we can build them if we have use of the axiom of choice. This might seem like a reason not to use the axiom of choice, but remember that Zorn’s lemma — which is equivalent to the axiom of choice — was essential when we needed to show that every vector space has a basis, or Tychonoff’s theorem, or that exact sequences of vector spaces split. So it’s sort of a mixed bag. In practice, most working mathematicians seem to be willing to accept the existence of non-Lebesgue measurable sets in order to gain the above benefits.

So, first a lemma: if \xi is irrational, then the set A of all numbers of the form n+m\xi with n and m any integers is dense in the real line. That is, every open interval U contains at least one point of A. The same is true for the set B where we restrict n to be even, and for the set C where we restrict n to be odd. Note A (and, incidentally, B) is actually a subgroup of the additive group of real numbers.

For every integer i there is some unique integer n_i so that 0\leq n_i+i\xi<1; we will write x_i=n_i+i\xi. If U is an open interval, there is some positive integer k with \mu(U)>\frac{1}{k}. Picking out the first k+1 numbers x_1,\dots,x_{k+1}, there must be some pair x_i and x_j with \lvert x_i-x_j\rvert<\frac{1}{k} (or else they wouldn’t all fit in the interval \left[0,1\right)). But then some multiple of x_i-x_j must land within U, as we asserted. For B, we can do the same using the interval \left[0,2\right), and for C we can use the fact that C=B+1.

Now, I say that there exists at least one set E_0 which is not Lebesgue measurable. To show this, we consider the quotient group \mathbb{R}/A. That is, we use an equivalence relation x\sim y if x-y\in A. This divides up the real numbers into a disjoint union of equivalence classes under this relation, and the axiom of choice allows us to build a set E_0 by picking exactly one point from each equivalence class. This is the set we will show is not Lebesgue measurable.

Suppose F is a Borel set contained in E_0. The difference set D(F) contains no point of A, since if this happened we’d have two points in E_0 picked from the same \sim-equivalence class. But we just saw that any open interval contains a point of A, and so our result from last time shows that F must have outer measure zero — if it had positive outer measure then D(F) would contain an open interval. And so if E_0 is Lebesgue measurable then its Lebesgue measure must be zero.

Now if a_1 and a_2 are distinct elements of A, then E_0+a_1 and E_0+a_2 must be disjoint. As we let a range over the countable number of values in A, the sets E_0+a then form a countable disjoint cover of \mathbb{R}. But each of the E_0+a is just a translation of E_0, and so each one must have the same measure. And then since Lebesgue measure is countably additive, we must have

\displaystyle\bar{\mu}\left(\mathbb{R}\right)=\bar{\mu}\left(\bigcup\limits_{a\in A}E_0+a\right)=\sum\limits_{a\in A}\bar{\mu}(E_0+a)=\sum\limits_{a\in A}0=0

But this is clearly nonsense.

We can do even better, actually, in our efforts to find bizarre sets. There exists a subset M\subseteq\mathbb{R} so that for every Lebesgue measurable set E we have both

\displaystyle\mu_*(M\cap E)=0
\displaystyle\mu^*(M\cap E)=\bar{\mu}(E)

That is, no matter what Lebesgue measurable set we pick, its intersection with M is so weird that no set of positive Lebesgue measure can fit inside it, and yet E itself is the smallest Lebesgue measurable sets that can contain it.

To find this set, write A=B\cup C from our lemma and take E_0 to the set we just constructed. Define M=E_0+B — the set of sums of points in E_0 and points in B. If F is a Borel set contained in M, then D(F) can’t contain any point of C (using a similar argument to that from earlier). And so we must have \mu_*(M)=0.

On the other hand, we just saw that E_0+A=\mathbb{R}, and thus

\displaystyle M^c=E_0+C=E_0+(B+1)=M+1

And so \mu_*(M^c)=0 as well. If E is any Lebesgue measurable set, the monotonicity of \mu_* gives us \mu_*(M\cap E)=\mu_*(M^c\cap E)=0. And then an earlier result tells us that

\displaystyle\bar{\mu}(E)=\mu_*(M^c\cap E)+\mu^*((M^c)^c\cap E)=0+\mu^*(M\cap E)=\mu^*(M\cap E)

April 24, 2010 - Posted by | Analysis, Measure Theory

7 Comments »

  1. You say that all three sets (A, B, C) are subgroups. I see that for A (all n) and B (all even n). But how is C (all odd n) an additive subgroup — it’s not even an additive group since it doesn’t contain 0.

    Or do I totally misunderstand?

    Thanks!

    Comment by marshall | April 25, 2010 | Reply

  2. Sorry, I misspoke in my haste to get out the catch-up post…

    Comment by John Armstrong | April 25, 2010 | Reply

  3. In case anyone is interested, the example you ended with is sometimes called a “maximally non-measurable set” or a “saturated nonmeasurable set”, and more examples and their properties can be found in the following posts:

    Remarks on Bernstein sets
    http://tinyurl.com/29sgs2b
    http://tinyurl.com/27sqz2d [minor correction]

    Now for something REALLY STRANGE. In 1917 Lusin and Sierpinski showed that the unit interval [0,1] can be partitioned into c = 2^(aleph_0) many pairwise disjoint sets each having Lebesgue outer measure 1. This shows a massive failure of additivity in the case of Lebesgue outer measure! And yes, they constructed c many such sets, not just uncountably many (which would give you “only” aleph_1 many such sets under CH).

    Here’s their paper:

    Nikolai N. Lusin and Waclaw Sierpinski, “Sur une décomposition d’un intervalle en une infinité non dénombrable d’ensembles non mesurables” [On a decomposition of an interval into a nondenumerably many nonmeasurable sets], Comptes Rendus Académie des Sciences (Paris) 165 (1917), 422-424.
    [JFM 46.0294.01] [available on-line]
    http://www.emis.de/cgi-bin/JFM-item?46.0294.01

    Comment by Dave L. Renfro | April 26, 2010 | Reply

  4. […] is), this condition reduces to asking that . If, further, , then we ask that . As an example, the maximally nonmeasurable set we constructed is […]

    Pingback by Measurable Subspaces II « The Unapologetic Mathematician | April 28, 2010 | Reply

  5. […] to add the requirement that . Also, the converse of this theorem is definitely not true; if is a non-measurable set, then the function is not measurable even though the absolute value is […]

    Pingback by Composing Real-Valued Measurable Functions I « The Unapologetic Mathematician | May 4, 2010 | Reply

  6. […] now we can take a thick, non-Lebesgue measurable set whose intersection with is itself a non-Lebesgue measurable set . However, , and has Lebesgue […]

    Pingback by Composing Real-Valued Measurable Functions II « The Unapologetic Mathematician | May 5, 2010 | Reply

  7. […] During this long period from 1904 to 1962, many bizzare results such as the Banach Tarsky paradox, existence of non-Lebesgue measurable sets etc were found. This put a suspicion in people’s minds as to whether this axiom was […]

    Pingback by The axiom of choice | Notes on Mathematics | September 19, 2012 | Reply


Leave a reply to Measurable Subspaces II « The Unapologetic Mathematician Cancel reply